Hydrostatic Force and Center of a Mass - Calc 2

Click For Summary
The discussion revolves around two main problems: finding the centroid of a region bounded by specific curves and calculating hydrostatic force on a vertical steel plate. For the centroid problem, it is suggested that the user may be working with the wrong region and should consider using two integrals due to the changing upper boundary. In the hydrostatic force problem, the user calculated pressure and force but received an incorrect answer, prompting a request for clarification on the setup. The importance of sketching the region and understanding the boundaries is emphasized for both problems. Further assistance is requested to clarify the integration process and resolve the hydrostatic force calculation.
Keldroc
Messages
8
Reaction score
0
Hey guys, I have a couple of problems about hydrostatic force and center of a mass that I was hoping someone could help me out with.

Center of mass

Homework Statement


Sketch the region bounded by the curves y=2x^3, x+y=3 and y=0. Find the coordinates of the centroid.

Homework Equations


Sorry I don't know how to make equations appear on here so I'll just type them out:
M=int f(x)-g(x)dx from a to b
x bar= 1/M * int x[f(x)-g(x)]dx from a to b
y bar= 1/M * int (1/2)[f(x)^2-g(x)^2]dx from a to b

The Attempt at a Solution


I used y=3-x for f(x) and y=2x^3 for g(x) and plugged them into the equations, used a=0 and b=3 and I got the wrong answer so I'm not sure what to do now.
Hydrostatic Force

Homework Statement


Find the following questions at the depth of 25m. The gravitational acceleration is g=9.8m/s^2 and the density of water is =1000kg/m^3:

Find the hydrostatic force on a square steel plate with sides 3m placed vertically.

Homework Equations


Pressure=\rhogd
F=Pressure * Area

The Attempt at a Solution


For pressure I did P=(1000kg/m^3)(9.8m/s^2)(25m)= 245000

Then, did Force=(245000 Pa)(3m * 3m)=2205000 N and it was the wrong answer.
 
Last edited:
Physics news on Phys.org
Keldroc said:
Hey guys, I have a couple of problems about hydrostatic force and center of a mass that I was hoping someone could help me out with.

Center of mass

Homework Statement


Sketch the region bounded by the curves y=2x3 x+y=3 and y=0. Find the coordinates of the centroid.


Homework Equations


Sorry I don't know how to make equations appear on here so I'll just type them out:
M=int f(x)-g(x)dx from a to b
x bar= 1/M * int x[f(x)-g(x)]dx from a to b
y bar= 1/M * int (1/2)[f(x)^2-g(x)^2]dx from a to b


The Attempt at a Solution


I used y=3-x for f(x) and y=2x^3 for g(x) and plugged them into the equations, used a=0 and b=3 and I got the wrong answer so I'm not sure what to do now.
I believe you are working with the wrong region. Did you sketch a graph of it? If you use vertical strips to calculate the area/mass, you will need two integrals, because the upper boundary changes at (1, 2). The region you should be working with has a sort of triangular shape (but with one curved side) and is bounded on its left side by y = 2x2 and on its right side by y = -x + 3.
Keldroc said:
Hydrostatic Force

Homework Statement


Find the following questions at the depth of 25m. The gravitational acceleration is g=9.8m/s^2 and the density of water is =1000kg/m^3:

Find the hydrostatic force on a square steel plate with sides 3m placed vertically.


Homework Equations


Pressure=\rhogd
F=Pressure * Area


The Attempt at a Solution


For pressure I did P=(1000kg/m^3)(9.8m/s^2)(25m)= 245000

Then, did Force=(245000 Pa)(3m * 3m)=2205000 N and it was the wrong answer.
 
Mark44 said:
I believe you are working with the wrong region. Did you sketch a graph of it? If you use vertical strips to calculate the area/mass, you will need two integrals, because the upper boundary changes at (1, 2). The region you should be working with has a sort of triangular shape (but with one curved side) and is bounded on its left side by y = 2x2 and on its right side by y = -x + 3.

Thanks for the response but I don't quite understand what you mean by setting up two integrals? Could you explain it to me?

Also I could still use help for the second problem. Thanks alot!
 
Question: A clock's minute hand has length 4 and its hour hand has length 3. What is the distance between the tips at the moment when it is increasing most rapidly?(Putnam Exam Question) Answer: Making assumption that both the hands moves at constant angular velocities, the answer is ## \sqrt{7} .## But don't you think this assumption is somewhat doubtful and wrong?

Similar threads

Replies
3
Views
2K
Replies
3
Views
2K
Replies
2
Views
2K
  • · Replies 10 ·
Replies
10
Views
2K
  • · Replies 3 ·
Replies
3
Views
2K
  • · Replies 1 ·
Replies
1
Views
1K
Replies
20
Views
2K
  • · Replies 21 ·
Replies
21
Views
2K
Replies
1
Views
6K
  • · Replies 10 ·
Replies
10
Views
5K